0
$\begingroup$

I am considering the discrete-time discrete-valued random process $X_n$ that consists of the sequences

Sequence1:   0 0 1 0 0 1 0 0 1 0 0 1 0 0 ... Sequence2:   1 0 0 1 0 0 1 0 0 1 0 0 1 0 ... Sequence3:   0 1 0 0 1 0 0 1 0 0 1 0 0 1 ... 

with each sequence being chosen with equal probability $\frac{1}{3}$.

After determining that this process is not an i.i.d. random process, I am having trouble finding the joint PMF of $X_{n_1}$ and $X_{n_2}$. Any help would be greatly appreciated. Thanks!

  • 0
    See also http://math.stackexchange.com/questions/258588.2012-12-14

0 Answers 0